LSAT and Law School Admissions Forum

Get expert LSAT preparation and law school admissions advice from PowerScore Test Preparation.

User avatar
 Dave Killoran
PowerScore Staff
  • PowerScore Staff
  • Posts: 5852
  • Joined: Mar 25, 2011
|
#87410
Complete Question Explanation
(The complete setup for this game can be found here: lsat/viewtopic.php?f=149&t=16450)

The correct answer choice is (C)

The condition in the question stem creates an L :longline: M sequence, which can be added to the G :longline: L :longline: F sequence and the M :longline: F inference:

PT58-Sept2009 LGE-G1-Q6-D1.png

Under this configuration, M cannot be begun in 601, and so G and S must be begun in 601:

PT58-Sept2009 LGE-G1-Q6-D2.png

Because L must be earlier than F, M, and H, we can deduce that L cannot be begun in 603, and so L must be begun in 602. Because M must have been begun earlier than 604, M must have been begun in 603. F and H then create a dual-option on 604 and 605:

PT58-Sept2009 LGE-G1-Q6-D3.png

Accordingly, answer choice (C) could be true and is correct.
You do not have the required permissions to view the files attached to this post.

Get the most out of your LSAT Prep Plus subscription.

Analyze and track your performance with our Testing and Analytics Package.